¿Cómo se prueba la segunda ley de la termodinámica a partir de la mecánica estadística?

¿Cómo se prueba la segunda ley de la termodinámica a partir de la mecánica estadística? ¿Para demostrar que la entropía solo aumentará con el tiempo? ¿Cómo probar? Por favor guía.

Esto es casi un duplicado de esta pregunta: physics.stackexchange.com/q/20401
También muy cerca del duplicado de physics.stackexchange.com/q/10690 y relacionado con physics.stackexchange.com/q/63416
@outrageous Perdón por mencionar un hilo antiguo, pero usted y los futuros lectores pueden encontrar este documento interesante. De ninguna manera soy un experto en esta área, por lo que, dadas las discusiones a continuación, sería correcto estar cansado. Prueba mecánica estadística de la segunda ley de la termodinámica basada en la entropía del volumen - Michele Campisi [enlace] sciencedirect.com/science/article/pii/S1355219807000974
@BenCrowell Consulte el enlace anterior. ¿Contrasta el pensamiento científico principal y el artículo que vinculó?

Respuestas (7)

¿Cómo se prueba la segunda ley de la termodinámica a partir de la mecánica estadística?

no puedes Para probar la segunda ley, que es asimétrica en el tiempo, necesitas algún ingrediente que rompa la simetría de inversión temporal. La mecánica estadística no tiene tal ingrediente. Para eliminar esta simetría, necesita condiciones de frontera asimétricas en el tiempo o leyes de la física asimétricas en el tiempo (Callender 2011). En ausencia de cualquiera de estos ingredientes, tienes la paradoja de Loschmidt : para cualquier sistema A que evoluciona de t 1 a t 2 para aumentar la entropía de S 1 a S 2 , podemos construir un sistema A que comienza con las partículas en las posiciones que tenían en t 2 , pero con momentos opuestos. Entonces el sistema evolucionará de S 2 a S 1 .

Lo que se puede derivar únicamente de la mecánica estadística es una forma de la segunda ley que dice que si un sistema experimenta una gran fluctuación alejándose del equilibrio, entonces, en tiempos suficientemente grandes tanto antes como después, estará, con alta probabilidad, más cerca del equilibrio. (Callender 2011). Esto es realmente sólo una declaración de ergodicidad, es decir, que todos los estados son igualmente probables.

La interpretación estándar de la segunda ley hoy en día es que surge de condiciones de contorno asimétricas. Por razones que desconocemos, tuvimos un Big Bang de baja entropía.

Aquí hay otra pregunta que esta casi duplica. Escribí una respuesta allí que explica algunas de las ideas con más detalle, para un sistema de juguete específico.

Referencias

Callender, Craig, "Asimetría termodinámica en el tiempo", The Stanford Encyclopedia of Philosophy (edición de otoño de 2011), Edward N. Zalta (ed.), http://plato.stanford.edu/archives/fall2011/entries/time-thermo

Una baja entropía en el origen del Big Bang no explica, por sí sola, el continuo aumento de la entropía. También podría haber disminuido hasta una entropía aún más baja, por ejemplo, y luego volver a aumentar.
@Trimok: Indiqué una forma débil de la segunda ley en mi segundo párrafo. Esto se discute con más detalle en la apertura de la sección 2 del documento de Callender. Dada la forma débil de la segunda ley, más la condición inicial de baja entropía, sigue la forma fuerte de la segunda ley. La ruptura de la simetría por el Big Bang se analiza en la sección 2.3.
@Trimok: agregué un enlace a una respuesta diferente en la que deletreé algunas de las cosas sobre las que está preguntando.
@BenCrowell No estoy del todo convencido por esto. En particular, aunque nunca he encontrado un argumento convincente, tengo la esperanza de que alguien pueda (quizás lo haya hecho) o demuestre en el contexto de la mecánica estadística cuántica que la segunda ley surge como consecuencia de la evolución unitaria acoplada a una noción adecuada. /interpretación de la entropía (es decir, la entropía de von-Neumann) y algunos argumentos sobre el aumento de la entropía debido al aumento de las correlaciones entre los subsistemas cuánticos. Entiendo el argumento de la asimetría del tiempo, pero las declaraciones de no-go en física son notoriamente poco confiables.
Gran artículo, no había visto este antes a pesar de que la enciclopedia de Stanford es uno de mis sitios favoritos.
Su "forma débil" es lo que me gusta llamar la ley de "distribuciones puntiagudas" como en la sección "Un paseo aleatorio por el espacio de fase" en mi respuesta aquí . Pensé que podría estar interesado, si aún no lo había pensado, en dar el ejemplo de la distribución binomial en sus clases: es un ejemplo muy, muy simple y muestra la esencia de su "forma débil". Lo que me recuerda, ¿tiene algún material sobre la segunda ley en Light and Matter?
@joshphysics: Estoy seguro de que no encontrará ninguna prueba de este tipo, ya que no se ha encontrado ninguna en los 160 años transcurridos desde que se formalizó por primera vez la segunda ley, y el argumento de Loschmidt explica por qué. Los teoremas de no-go pueden no ser confiables en algunos casos, pero uno que ha resistido la prueba del tiempo durante 160 años me parece bastante confiable.
@WetSavannaAnimalakaRodVance: Lo que me recuerda, ¿tiene algún material sobre la segunda ley en Light and Matter? Aquí tienes: lightandmatter.com/html_books/0sn/ch05/ch05.html
Gracias. También tengo las reservas de joshphysics, pero el argumento de Loschmidt es extremadamente simple. ¡160 años no fuera también es una evidencia experimental bastante buena!
@joshphysics Para que quede claro, ¿prevé / espera un ataque a la simetría del tiempo o la simetría del tiempo seguirá en pie? El aumento de las correlaciones de los subsistemas suena como algo que encajaría en la "forma débil" de Ben. ¿Le gustan las obras de Jaynes, especialmente "Gibbs vs Boltzmann Entropy" ? Creo que he oído algo parecido a lo que usted prevé como un argumento para que el tiempo psicológico vaya en una dirección: nuestras mentes se vinculan cada vez más con el mundo exterior, pero estoy bastante seguro de que, para empezar, uno asumió una entropía baja.
@WetSavannaAnimalakaRodVance Es más un sueño ingenuo. Tengo un poco de familiaridad con el trabajo de Jaynes. Si fuera un hombre de apuestas (lo que prefiero ser), apostaría a que Ben tiene razón en esto, pero tengo una fuerte aversión a las afirmaciones de imposibilidad.
@joshphysics Estoy totalmente de acuerdo con los teoremas de imposibilidad en física. Estoy muy intrigado por ellos en matemáticas, por lo que me atraen en física, pero los considero más como "evidencia experimental". Para mí en termodinámica, el experimento es la reina: me he frito el cerebro lo suficiente a lo largo de los años mirando "pruebas" y cosas por el estilo. No creo que tengamos una base de probabilidad lo suficientemente rigurosa como para "probar" mucho (aparte de las propiedades de medida y demás, que son solo modelos). Sonrío cuando la gente pregunta "¿por qué QM puede usar probabilidades clásicas?" - como, ¿crees que sería más fácil?
Si estuviera buscando una escapatoria o una forma de evitar el argumento de Loschmidt, un posible lugar para buscar sería algún tipo de ruptura de simetría espontánea. Posiblemente relevante: arxiv.org/abs/1011.4173v5
@joshphysics Estoy estudiando para mis finales orales de pregrado y usé muchos de mis últimos días tratando de encontrar una justificación adecuada para la minimización de tr ( ρ Iniciar sesión ρ ) en física cuántica estadística. Hablé con mi examinador sobre mi frustración y dijo que espera que algún día haya un argumento convincente que solo se base en axiomas de la mecánica cuántica y no necesite más información. Me parece fascinante cómo esto no se resuelve... Quiero decir que es tan importante. Es el bloque de construcción para toda la ingeniería mecánica, aunque por lo general no cavan tan profundo.
@joshphysics de todos modos, estaba sugiriendo un argumento muy similar al que estabas hablando en tu comentario. Pero también dijo que no todos creen que tal argumento sea posible. Es tan insatisfactorio que necesitamos un axioma adicional (minimización de la entropía) para que la física estadística funcione.
@TheoreticalMinimum Probablemente le interese lo siguiente y las respuestas que contiene: physics.stackexchange.com/q/63416/19976 , physics.stackexchange.com/q/53147/19976
@joshphysics el segundo enlace me hizo reír. Es exactamente esta cita de sakurai la que me hizo preguntarme sobre esto en primer lugar. casualmente deja caer esta declaración como si fuera una prueba menor dejada como ejercicio, sin siquiera dar ninguna referencia. ¡Gracias! Voy a echar un vistazo al papel de Jaynes algún día. ¿Puede decir qué tan satisfecho ha estado con su explicación del 1 al 10?
@TheoreticalMinimum Sí, es una pequeña nota al margen bastante cómica en Sakurai. Cuando leí a Jaynes por primera vez, me quedé impresionado, pero estoy cada vez más insatisfecho cuanto más lo pienso. Actualmente no estoy seguro de qué hacer con eso muchos años después. Mi sensación actual es que su explicación arroja algo de luz sobre el tema desde la perspectiva de la teoría de la información, pero ignora la dinámica microscópica y cómo la termodinámica y la maximización de la entropía pueden surgir o no de esa dinámica, una pregunta en la que los físicos están bastante interesados.

Un teorema relevante aquí que parece (además y separado de los argumentos de la paradoja de Loschmidt mencionados por Ben Crowell ) pesar en contra de una prueba de la segunda ley es el Teorema de recurrencia de Poincaré que, en términos generales, un sistema (con ciertas suposiciones) hará, dado el tiempo suficiente, evolucionar de nuevo a algo arbitrariamente cerca de su estado inicial. Más precisamente, citando la declaración en Wikipedia.

Dejar ( X , Σ , m ) sea ​​un espacio de medida finita y sea F : X X ser una transformación que conserva la medida. ...

Teorema :

Para cualquier mi Σ , el conjunto de esos puntos X de mi tal que F norte ( X ) mi para todos norte > 0 tiene medida cero. Es decir, casi todos los puntos de mi vuelve a mi . De hecho, casi todos los puntos regresan infinitamente a menudo; "es decir"

m ( { X mi :  existe  norte  tal que  F norte ( X ) mi  para todos  norte > norte } ) = 0.

o, de manera informal, la medida del conjunto de puntos en el espacio de fases que en algún momento no son mapeados a sí mismos por la evolución del sistema no tienen medida, o "casi no hay puntos que no sean mapeados a sí mismos por alguna evolución de el sistema a lo largo del tiempo".

Entonces, ¿cómo aplicamos esto al Universo? Necesitamos algunas suposiciones.

  1. El espacio de fases del Universo X es un concepto significativo y puede interpretarse como un espacio de medida finita , es decir (i) podemos definir un σ -álgebra y una medida para ella (ii) X es la unión contable de conjuntos medibles con medida finita;
  2. La medida en 1. se conserva por las leyes de la física. Las personas que creen en este argumento a favor del Universo suelen considerar que esto es cierto, porque interpretan la medida en 1. como la medida del volumen de fase y luego el teorema de Liouville (consulte la página Wiki con este nombre) asegura que se conserva. Por lo tanto, debemos asumir el teorema de Liouville.

Entonces, en términos generales, se puede encontrar un límite superior finito en el volumen de fase "accesible". Si el Universo resulta ser finito espacialmente, entonces esto sería razonable, y se cumple el teorema de Liouville.

Entonces, dadas ciertas suposiciones que suenan razonables sobre el Universo, una prueba de la segunda ley de la termodinámica es una esperanza vana, porque con el tiempo suficiente, el Universo volverá a un estado de entropía que tenía en el pasado.

Por supuesto, las suposiciones muestran que hay varias formas de fallar en este argumento, pero una prueba de la segunda ley de la termodinámica iría en contra de al menos una de las suposiciones 1. y 2., por lo que tendría implicaciones interesantes para otra física. , modelos cosmológicos permitidos y cómo funcionan sus espacios de fase en particular.

Oliver Penrose aborda el teorema de recurrencia de Poincaré al observar que los tiempos de recurrencia dependen de las condiciones iniciales y que el sistema podría regresar inmediatamente cerca del origen o después de la edad del universo. Estado de equilibrio. mecánico se ocupa de eso al permitir todos los microestados posibles (incluso aquellos que uno ingenuamente caracterizaría como "falta de equilibrio" como todas las partículas en la esquina de una caja).
@gatsu Gracias por el enlace. No he leído a Oliver Penrose, solo a su famoso hermano, pero está en la lista de lecturas pendientes. ¿Está su discusión en la serie de conferencias en el enlace web que me enviaste? Sí, estoy de acuerdo en que probablemente esté hablando de tiempos fantásticamente largos para que ocurra la recurrencia, pero si va a demostrar que la segunda ley es consistente con el PRT, entonces tendría que ser una ley que diga (i) la entropía es casi seguro que aumentará en la escala de tiempo T y (ii) dar alguna estimación de T . Por supuesto, también como en mi respuesta, todas las apuestas están canceladas si el Universo es patológicamente infinito en amplitud.
De acuerdo con los avances recientes en la física fuera del equilibrio, lo que importa podría ser la variación de entropía promedio y no la entropía real para un proceso dado. Esta ya era la esencia de las respuestas de Boltzmann a las objeciones de Loschmidt y Zermelo. Mi punto anterior era simplemente decir que lo que importa es que existe una distribución de probabilidad invariante que maximiza una entropía funcional y hacia la cual tenderán casi todas las distribuciones iniciales en el límite de tiempo infinito.
@gatsu Creo que estoy de acuerdo con usted "lo que más importa ..." - ¿es esto lo mismo que las "distribuciones puntiagudas" en la sección "Un paseo aleatorio por el espacio de fase" de mi respuesta aquí ? Que esencialmente hay estados de máxima entropía y no mucho más, por lo que el sistema siempre tenderá a estar en estos estados con mayor frecuencia a fuerza de la "caminata aleatoria". Esto, por supuesto, también depende de la hipótesis ergódica, que no será estrictamente cierta en todos los casos debido a las correlaciones entre microestados, pero, como hablo en mi respuesta, hay intuitivas ...
..argumentos de que todavía se mantiene en un nivel apropiado de granulado grueso, por lo que las correlaciones no importarán. Ahora, todo esto es algo muy antiguo (años 50 y 60), esencialmente el trabajo de ET Jaynes (de cuyos artículos aprendí la mayor parte de mi termodinámica), por lo que es mucho antes de los teoremas de Searle (o Bernhardt) y compañía, que he glosado pero aún no ha atravesado profundamente. De todos modos, estoy de acuerdo en que la distribución "promedio" debe ser máxima, pero ¿eso no solo dice que la recurrencia de Poincaré solo sacará al sistema de este estado por fracciones fugazmente cortas de tiempo a largo plazo?
@WetSavanna...: Creo que mi punto es diferente al de la interpretación de distribución puntiaguda del segundo principio de la termodinámica. La razón es porque dentro de su interpretación, el teorema de recurrencia de Poincaré sigue siendo un problema, mientras que en el que expuse no lo es. Creo que lo que importa es mirar la distribución de probabilidad y no los sistemas individuales. En ese caso, el problema de recurrencia de Poincaré se convierte simplemente en un ruido estadístico que aparece en la distribución de probabilidad cuando el tiempo tiende a infinito.

Si considera el postulado de la probabilidad a priori igual, dará la conclusión al mismo tiempo: tenderá a la distribución más probable. En mecánica estadística, tender a la distribución más probable es una probabilidad, y para la entropía de Boltzmann, d S 0 es también una probabilidad pero no un resultado inevitable. Así que no puedes probar d S 0 como resultado inevitable de la mecánica estadística.

Por otro lado, el postulado de igual probabilidad a priori no necesita ser considerado para la termodinámica, por favor considere la termodinámica local de no equilibrio, en la ecuación

d i S d t = i j i · X i 0 ,

algunas de las fuerzas motrices X i de los procesos irreversibles no se originan de la condición de igual probabilidad a priori, como el gradiente de fuerza generalizada X i = Y , el gradiente de potencial químico X i = m j , por lo que la demostración de la segunda ley de la termodinámica a partir de la mecánica estadística estará incompleta.

Esta pregunta es irrelevante para la simetría T de la física. Las leyes T-simétricas y las leyes T-asimétricas son las dos leyes diferentes, las dos describen principios diferentes de la física. El punto clave es que las estructuras teóricas de la termodinámica, la mecánica estadística y la dinámica son diferentes. Como es bien sabido, la primera ley de la termodinámica es también una ley T-simétrica.

d tu = d q + d W + j m j d norte j

Dudar de la segunda ley de la termodinámica por la T-simetría de las primeras leyes no tiene sentido, debido a que las dos involucran diferentes principios de la física, y de manera similar, tampoco podemos dudar de la segunda ley de la termodinámica por las leyes T-simétricas de la dinámica. Las leyes de la dinámica simétricas en el tiempo deben compararse con la primera ley de la termodinámica, pero no con la segunda ley.

¿Cómo se prueba la segunda ley de la termodinámica a partir de la mecánica estadística? ¡ y la prueba matemática de la segunda ley de la termodinámica son dos preguntas diferentes!

La Segunda Ley de la Termodinámica es una aproximación, tiene validez estadística o probabilística. La Mecánica Estadística corrige la versión simple que dice que la entropía nunca disminuye a lo siguiente.

La gran mayoría de las veces, un sistema lo suficientemente grande que no es un sistema cerrado (en el sentido de la mecánica: tenga en cuenta que en Termodinámica, la frase "sistema cerrado" tiene un significado diferente al que tiene en la mecánica hamiltoniana) pero es en termal contacto con su entorno, si pasa de un estado de equilibrio a otro, no disminuirá su entropía.

Ahora, la entropía en el sentido de la termodinámica ni siquiera está definida para estados que no son estados de equilibrio. Si el Universo como un todo no está en un estado de equilibrio, no posee una entropía bien definida. Eso va el doble de su condición inicial. La última vez que miré el Universo, no parecía que estuviera en equilibrio. Entonces, la abrumadora evidencia empírica de la verdad de la Segunda Ley de la Termodinámica no dice nada sobre el Universo.

Una excelente discusión del tema entre Zermelo (y Loschmidt) y Boltzmann sobre el teorema H frente a la reversibilidad y la recurrencia de Poincaré se encuentra en von Plato, Creating Modern Probability, y por Janneke van Lith (2001). Ergodic Theory, Interpretations of Probability and the Foundations of Statistical Mechanics, un excelente artículo de revisión. Debido a las lagunas de la "probabilidad", las objeciones de Zermelos a la interpretación ingenua de la Segunda Ley no se aplican a la versión estadística eventualmente más matizada de Boltzmann.

La tesis doctoral del Dr. van Lith es de acceso abierto http://dspace.library.uu.nl/bitstream/handle/1874/657/full.pdf?sequence=1

al igual que su reseña del misch-masch de Guttmann http://www.projects.science.uu.nl/igg/dis/guttmann.html

Primero, introduzcamos un pequeño subsistema de sistema aislado. El número norte partículas de este subsistema es suficientemente grande para interpretar el subsistema como casi cerrado (las fluctuaciones de los valores macroscópicos son proporcionales a 1 norte ). Entonces podemos decir que la función de distribución, según el teorema de Liouville, es integral del movimiento del subsistema. Entonces es posible decir que la función de distribución en este caso es función de energía. Entonces, para la distribución de energía del subsistema, es posible escribir

d PAGS mi = mi mi + d mi ρ ( mi ) d Γ 2 norte = ρ ( mi ) mi mi + d mi d Γ 2 norte = ρ ( mi ) d Γ mi = ρ ( mi ) d Γ mi d mi d mi = ρ mi ( mi ) d mi , ( .1 )
dónde d Γ 2 norte es el elemento del volumen de fase del subsistema ( 2 norte Referirse a 6 norte , dónde norte ),

ρ ( mi ) es una función de microdistribución de energía (es decir, posibilidad de encontrar un subsistema en un estado con energía mi , mi + d mi que corresponde a algún impulso pags , pags + d pags del elemento (!) del volumen de fase), que es "casi" constante en el elemento Γ 2 norte , por lo que podemos llevarlo fuera de la integral,

d Γ mi = mi mi + d mi d Γ 2 norte se refieren al valor de la capa "esférica" ​​que se refiere a los impulsos pags , pags + d pags de subsistema.

ρ mi ( mi ) es la función de la macrodistribución de energía (es decir, encontrar un subsistema en un estado con energía mi , mi + d mi a la que corresponden todos los impulsos posibles pags , pags + d pags de volumen de fase).

Entonces, la energía de un sistema cuasicerrado dado es casi constante y se encuentra en un vecindario pequeño Δ mi valor de energía cercano al promedio mi . Esto conduce al pico de la función de distribución para mi = mi . Esto significa que

ρ mi ( mi ) Δ mi 1 .
Al volver a ( .1 ) hace posible escribir
ρ ( mi ) Δ Γ mi 1 ,
dónde Δ Γ mi corresponde al elemento (no infinitesimalmente pequeño) del volumen de fase en el que el subsistema pasa la mayor parte del tiempo. Por lo tanto, consiste en información sobre el número total de estados microscópicos de un subsistema, que crean su estado macroscópico con energía. mi . De este modo Δ Γ mi determina el peso estadístico Ω de subsistema:
Δ Ω ( mi ) = a Δ Γ mi .
Después Δ Ω ( mi ) puede representarse como producto de ω i ( mi i ) de subsubsistemas del subsistema:
Δ Ω ( mi ) = ω 1 ( mi 1 ) . . . ω metro ( mi 1 ) .
Entonces es conveniente usar el logarifmo de Δ Ω ( mi ) , que se llama la entropía S :
S = en ( Δ Ω ( mi ) ) = en ( a Δ Γ mi ) .
Volvamos a ( .1 ) . Función ρ ( mi ) cambia lentamente al compararlo con Δ Γ mi . Así que para estados macroscópicos
Δ PAGS mi Δ Γ mi = 1 a mi S .
Esto lleva a la siguiente declaración: la posibilidad de un estado macroscópico dado del sistema aumenta cuando aumenta la entropía. Entonces, para un gran subsistema, la posibilidad de pasar a un estado con menos entopía está fuertemente suprimida.

Luego solo queda establecer un vínculo entre la entropía y el calor y, finalmente, obtener la segunda ley de la termodinámica.

Lo siento, pero -1. Excepto por las últimas tres oraciones, esta respuesta solo establece que la probabilidad aumenta exponencialmente con el aumento de la entropía. Luego dice: "Esto lleva a la siguiente declaración: la posibilidad de un estado macroscópico dado del sistema aumenta cuando aumenta la entropía. Entonces, para un subsistema grande, la posibilidad de pasar a un estado con menos entopía está fuertemente suprimida". La segunda oración no se sigue de la primera; no puede, porque el primero es simétrico en inversión de tiempo y el segundo no lo es. Consulte en.wikipedia.org/wiki/Loschmidt%27s_paradox .
@BenCrowell. No entiendo por qué la segunda afirmación no es la consecuencia de la primera. Tengamos un sistema cerrado sin equilibrio con energía. mi . Podemos repartirlo en pequeños subsistemas casi cerrados con energías mi i , i mi i = mi . Los subsistemas pequeños alcanzan un equilibrio (que puede describirse como el estado de máxima entropía, que es consecuencia de una definición de equilibrio) más rápido que el sistema. Es decir, es posible un caso en el que hay un equilibrio en los subsistemas, pero no hay un equilibrio a lo largo de los subsistemas. Así que todo el sistema no está en equilibrio.
@BenCrowell. Pero la posibilidad d PAGS ( mi ) porque el sistema es el producto de posibilidades d PAGS i mi X pags ( S i ( mi i ) ) de pequeños subsistemas. Si el sistema no está en equilibrio, las energías mi i de sus subsistemas pueden interpretarse como energías variables, por lo que se establece el equilibrio. La mayor probabilidad se alcanza cuando S = S metro a X .
@BenCrowell. Entonces, si en el estado inicial (que corresponde al momento inicial del tiempo) el sistema cerrado no estaba en equilibrio (debe estar no cerrado antes del momento inicial, por supuesto), lo más posible es que vaya al estado con aumento monótono de la entropía, es decir, al estado de equilibrio. lleva a la declaración d S d t 0 .
@BenCrowell. No usé ecuaciones dinámicas; Solo usé la función de distribución. Por lo tanto, no hay nada contradictorio, por lo que la explicación no conduce a la paradoja de Loschmidt.
No puede derivar un resultado asimétrico en el tiempo a partir de suposiciones simétricas en el tiempo. No se establecen suposiciones asimétricas en el tiempo en ninguna parte de su argumento. Ni siquiera se refiere al tiempo o a una dirección del tiempo hasta la oración, "Hasta ahora, un gran subsistema ..." Cualquier sistema que evolucione en el tiempo como usted afirma puede revertirse en el tiempo para dar un sistema que tiene una entropía decreciente. Esta es exactamente la paradoja de Loschmidt.
@BenCrowell. La paradoja de Loschmid se refiere a la incompatibilidad de los resultados mecánicos y estadísticos sobre la simetría de la inversión del tiempo. "Mis" derivaciones no contienen una descripción mecánica del sistema, por lo que tampoco contienen "supuestos simétricos en el tiempo". No contiene ninguna suposición sobre la inversión del tiempo. Y finalmente, si el sistema está cerrado, el "camino de regreso" en el tiempo para él está deprimido. Si no es así, especifique dónde exactamente me equivoqué.
No contiene ninguna suposición sobre la inversión del tiempo. Nunca dije que lo hiciera. Señalé que carecía de suposiciones sobre el tiempo. Sin tales suposiciones, no se puede elegir una dirección del tiempo. Si no es así, especifique dónde exactamente me equivoqué. Hice esto en mi primer comentario. Señalé qué oración era la primera que no se seguía de la anterior.
@BenCrowell: Estoy de acuerdo contigo, por supuesto. Sin embargo, todavía es posible derivar un comportamiento irreversible de la dinámica reversible en límites adecuados, siempre que se excluyan conjuntos de condiciones iniciales de medida cero (lo que rompe la simetría de inversión de tiempo). Este es el caso de todas las derivaciones rigurosas de la ecuación de Boltzmann (ver, por ejemplo, el famoso trabajo de Lanford: Evolución temporal de grandes sistemas clásicos, o sus artículos en Comm. Math. Phys. 9 1968 176–191 y Comm. Math. Phys. 11 1968/1969 257–292).
Interesante. ¿Sabe dónde se presenta el argumento en algún lugar que no tiene un muro de pago? Me interesaría ver cómo se ve el conjunto excluido de condiciones iniciales. Supongo que también se podría excluir un conjunto idéntico de condiciones finales, en cuyo caso se obtendría el inverso de la segunda ley (entropía siempre decreciente).
@BenCrowell: No pude encontrar ninguno. Pero es posible que le guste el siguiente documento (que acabo de encontrar): igitur-archive.library.uu.nl/phys/2011-0316-200303/uffink.pdf . No lo he leído, pero parece discutir los resultados de Lanford en detalle. Tenga en cuenta también que hay muchas derivaciones más recientes de la ecuación de Boltzmann (bajo diferentes conjuntos de suposiciones), y debería poder encontrar algunas en arXiv. Un resultado reciente de este tipo que encuentro bastante notable es este: arXiv:1305.3397. Hay otras referencias interesantes allí.

Creo que una respuesta satisfactoria a su respuesta sería bastante extensa. Solo te doy un par de referencias.

En cuanto a la segunda ley de la termodinámica, se puede derivar de la física estadística a partir del supuesto del postulado de igual probabilidad previa, sobre el cual les sugiero leer el capítulo 1 del libro de Reifs: Mecánica estadística

Por lo que respecta al hecho de que la Entropía solo puede aumentar con el tiempo, le sugiero que eche un vistazo al llamado "teorema H" o "teorema de irreversibilidad", que puede encontrar explicado paso a paso aquí .

La respuesta de Trimok da un bosquejo del teorema H. El teorema H no es realmente una prueba de la segunda ley, por las razones dadas en mi comentario sobre la respuesta de Trimok. El autor (¿anónimo?) de las notas de OCW a las que se vinculó da una interpretación confusa de agitar las manos en las páginas 4-5 y, en mi opinión, no logra abordar el problema subyacente.

Una idea de la explicación, aunque no del todo rigurosa.

Bajo la hipótesis del caos molecular , se pueden considerar colisiones de dos cuerpos A B A B , y tenemos, para esta colisión particular:

d pags A d t = d pags B d t = d pags A d t = d pags B d t = C A , B , A , B ( pags A pags B pags A pags B )

dónde C A , B , A , B es una constante positiva.

Empezando con pags yo = 1 , la variación de la entropía S es d S d t = Iniciar sesión pags yo d pags yo d t

Entonces, obtenemos:

d S d t = 1 4 [ A ( Iniciar sesión pags A d pags A d t ) + B ( Iniciar sesión pags B d pags B d t ) + A ( Iniciar sesión pags A d pags A d t ) + B ( Iniciar sesión pags B d pags B d t ) ]

Eso es :

d S d t = 1 4 A , B , A , B C A , B , A , B ( Iniciar sesión pags A + Iniciar sesión pags B Iniciar sesión pags A Iniciar sesión pags B ) ( pags A pags B pags A pags B )

Finalmente :

d S d t = 1 4 A , B , A , B C A , B , A , B ( Iniciar sesión pags A pags B Iniciar sesión pags A pags B ) ( pags A pags B pags A pags B )

Porque el C son constantes positivas y yo o gramo es una función monótona ( X > y Iniciar sesión X > Iniciar sesión y ), la expresión anterior es positiva:

d S d t 0

Este argumento no es del todo correcto, como lo señala Loschmidt. No se puede derivar un resultado asimétrico en el tiempo sin condiciones de contorno asimétricas en el tiempo o leyes de la física asimétricas en el tiempo. Consulte en.wikipedia.org/wiki/Molecular_chaos , en.wikipedia.org/wiki/Loschmidt%27s_paradox y plato.stanford.edu/archives/fall2011/entries/time-thermo .